返回列表 发帖

请教大全-2-18


请教一下各位这道题:Lark Manufacturing Companyinitiated a voluntary Quality Circles program for machine operators.Independent surveys of employee attitudes indicated that the machine operatorsparticipating in the program were less satisfied with their work situationsafter two years of the program’s existence than they were at the program’sstart. Obviously, any workers who participate in a Quality Circles programwill, as a result, become less satisfied with their jobs.

Eachof the following, if true, would weaken the conclusion drawn above EXCEPT:
(A) The second survey occurred during aperiod of recession when rumors of cutbacks and layoffs at Lark Manufacturingwere plentiful.
(B) The surveys also showed that thoseLark machine operators who neither participated in Quality Circles nor knewanyone who did so reported the same degree of lessened satisfaction with theirwork situations as did the Lark machine operators who participated in QualityCircles.
(C) While participating in QualityCircles at Lark Manufacturing, machine operators exhibited two of the primaryindicators of improved job satisfaction: increased productivity and decreasedabsenteeism.
(D) Several workers at Lark Manufacturingwho had participated in Quality Circles while employed at other companiesreported that, while participating in Quality Circles in their previouscompanies, their work satisfaction had increased.E

(E) The machine operators who participatedin Quality Circles reported that, when the program started, they felt thatparticipation might improve their work situations.
答案选的是E. 可是我觉得B问题很大啊,这个题干的结论似乎是:Obviously, any workers who participate in a Quality Circles program will, as a result, become less satisfied with their jobs. B选项似乎说的是没参加的人,和结论无关,因此我觉得无法削弱啊,但为什么不选这个呢?
收藏 分享

The option E fits in because the machine operators had an expectation going into the program that it would help them improve their work situations. But if it did not fulfill their expectation, then it surely would be a cause of dissatisfaction. Hence option E is correct!

TOP

第一次做这道题,分析一下文章
P:有一个program,operator参加后,说自己对working situation的satisfaction下降,跟这个项目一开始想比较
C:参加这个program的workers对工作的satisfaction下降
题目要求Weaken
Obviously暗示文章是因果性结论,那么削弱的方法就有:找他因,资料有误,因果颠倒,有因无果,有果无因
B选项:说那些既没有参加也不知晓这个项目的员工也report了相同程度的dissatisfaction。——有果无因,削弱结论
就是说,虽然没有参加项目(无因),却也有这样的不满意程度(有果),暗示文章的结论存在推理上的错误

TOP

谢谢

明白了,多谢!

TOP

返回列表

站长推荐 关闭


美国top10 MBA VIP申请服务

自2003年开始提供 MBA 申请服务以来,保持着90% 以上的成功率,其中Top10 MBA服务成功率更是高达95%


查看